LSAT and Law School Admissions Forum

Get expert LSAT preparation and law school admissions advice from PowerScore Test Preparation.

User avatar
 Dave Killoran
PowerScore Staff
  • PowerScore Staff
  • Posts: 5853
  • Joined: Mar 25, 2011
|
#26532
Complete Question Explanation

The correct answer choice is (D)

The condition in the question stem suspends the third rule and replaces it with a GH block, creating the following chain:
October 97_M12_L3_explanations_game#1_#5_diagram_1.png
Since G and H form a block that cannot come before F due to space restrictions, the following setup results:
October 97_M12_L3_explanations_game#1_#5_diagram_2.png
Because J or K must come immediately ahead of F, they cannot be consecutive, and thus answer choice (D) is correct in this Cannot Be True question (Could Be True Except is identical to Cannot Be True).
You do not have the required permissions to view the files attached to this post.
 Marce
  • Posts: 27
  • Joined: Sep 05, 2016
|
#32091
Please explain why J and K are placed on the first spot, and why the explanation says J and K has to come immediately before F; I don't see this in the global rules nor in the local question itself.
 Adam Tyson
PowerScore Staff
  • PowerScore Staff
  • Posts: 5153
  • Joined: Apr 14, 2011
|
#32098
It's not in the rules, Marce, because it's an inference - something not explicitly stated that must nonetheless be true because of the actions of the rules,

In this case, if not J or K, what could be first? Never F, because it is always 2nd. Never L or M because they are always after H. Now, with G immediately before H, neither G nor H can be in the first position - H because it is after G, and G because F gets in the way (if G was first, H would have to be second, immediately after G, but it can't be because the rules still have F second). J and K are the only variables left that could possibly be 1st, so one of them must be!

Work on making those inferences - they are the key to success in LG. They can be as simple as a not-law (like L cannot ever be first in this game - not in the rules, but because of the rules) and they can be as complicated as the G4 :arrow: H1 inference we spoke about in another post.

Good luck!
 Marce
  • Posts: 27
  • Joined: Sep 05, 2016
|
#32425
The rules on pg 3-20 of the Powerscore Test Prep states "G CAN COME NEITHER IMMEDIATELY BEFORE NOR IMMEDIATELY AFTER H, that's not the same as coming immediately before; I don't know what rules you are referring to.
Please advise.
 Adam Tyson
PowerScore Staff
  • PowerScore Staff
  • Posts: 5153
  • Joined: Apr 14, 2011
|
#32442
Hey again, Marce! For this question we are told to suspend that original rule about G and H not being adjacent, and replace it with a rule that says G is, just for this question, immediately before H, forming a GH block as shown in our explanation. For local questions, be sure to apply the local restrictions given in the question stem.

Get the most out of your LSAT Prep Plus subscription.

Analyze and track your performance with our Testing and Analytics Package.